Answer:
Step-by-step explanation:
Step 1
1 of 2
\textbf{Given}Given
n_{w}=1.33n
w
=1.33
$\theta_w=31\text{ ^\circ
∘
}$
$\theta_b=27\text{ ^\circ
∘
}$
\textbf{Approach}Approach
In this problem we are going to use Snell's law.
\textbf{Solution}Solution
The definition of Snell's law of refraction is
\begin{align} {n_1\cdot\sin \theta_1}={n_2\cdot \sin \theta_2} \end{align}
n
1
⋅sinθ
1
=n
2
⋅sinθ
2
where indexes 11 and 22 represent two different mediums. Since the motion is in the water we write
\begin{align} &{n_{w}\cdot\sin\theta_{w}}={n_{b}\cdot\sin\theta_{b}} \\ &{n_b}={n_{w}\cdot \frac{\sin\theta_{w}}{ \sin \theta_{b}} } \\ &{n_b}={1.33\cdot \frac{\sin31^\circ}{ \sin27^\circ} } \\ &\boxed{{n_b}=1.5} \end{align}
n
w
⋅sinθ
w
=n
b
⋅sinθ
b
n
b
=n
w
⋅
sinθ
b
sinθ
w
n
b
=1.33⋅
sin27
∘
sin31
∘
n
b
=1.5